Follow Slashdot stories on Twitter

 



Forgot your password?
typodupeerror
×
Wireless Networking Science

'Twisted' Waves Could Boost Capacity of Wireless Spectrum 147

New submitter Ogi_UnixNut writes "In Venice, Italy, physicists have shown that it is possible to use two beams of incoherent radio waves, transmitted on the same frequency but encoded in two different orbital angular momentum states, to simultaneously transmit two independent radio channels. In principle this allows the implementation of an infinite number of channels in a given, fixed bandwidth, even without using polarization, multiport or dense coding techniques. It's potentially a boon for congested spectrum problems, although at the moment I suspect it would only work for directional links."
This discussion has been archived. No new comments can be posted.

'Twisted' Waves Could Boost Capacity of Wireless Spectrum

Comments Filter:
  • Multipath (Score:5, Insightful)

    by rullywowr ( 1831632 ) on Friday March 02, 2012 @09:54AM (#39219723)
    What about the issue of multipath, where one wave inverses the phase because its reflection arrives at the antenna slightly delayed from the original direct LOS (line of sight) signal?
    I work with wireless microphones and deal with spectrum issues on a daily basis. With the shrinking spectrum, this would be extremely good news if it actually was feasible and practical in the real world. As it stands right now, two transmitters operating on the same frequency is simply a recipe for disaster.
    oh yeah, first!
    • by Forty Two Tenfold ( 1134125 ) on Friday March 02, 2012 @10:33AM (#39219983)

      Leeloo Dallas multipath!

    • by tibit ( 1762298 )

      GPS transmitters on the space segment all share the same channels and it's not a recipe for a disaster. Your GPS receiver works just fine listening to all those satellites all jabbering on the same channels. You engineer the whole system for it. Let's put it this way: wireless microphones are not anywhere near state-of-the-art in digital data transmission techniques. Extraterrestrial links are where the state of the art is at, and mostly has been, too, for a good while.

      • Hmmmm, so now we have a way to take those companies that have gobbled up most of the U.S. broadcast frequencies, and have them put all of their operations in one city on just one channel? Sounds great. Let's do it tomorrow.

        Actually with that HD radio technology that nobody but them seem to want (proprietary codec and all), we've already got a way to tell them to put everything in one r.f. channel. Maybe we can bring back some locally owned diverse broadcasting to the U.S.

        http://www.engineeringradio.us/bl [engineeringradio.us]

      • Re:Multipath (Score:5, Insightful)

        by YoopDaDum ( 1998474 ) on Friday March 02, 2012 @01:32PM (#39221795)
        That's completely different. GPS uses CDMA, which is a way to multiplex several users on the same channel. Here it's a way to create additional independent channels. The former is sharing one channel capacity, the later is adding channels and capacity. If you want to compare this to an existing technology, it's closer in spirit to MIMO with spatial multiplexing.

        But as the grand-parent remarked, and if I understand correctly, this shouldn't be robust to multipath (i.e. all the reflections that adds up at the receiver). And all practical use cases you care about as an end user must support multipath (OFDMA used in WiMAX and LTE main strength is its robustness to multipath) as they must operate in non line of sight (NLOS) conditions. So that would limit the application to line of sight (LOS) systems like microwave trunking. Possibly still useful, but not for you and me.

        And by the way, although you're correct that wireless microphones are basic tech, satellites links are by no mean state of the art. Satellite is LOS, the challenge is very low signal level but the channel is easy. The state of the art is in terrestrial broadband (mostly LTE and its evolutions now) with mobility and multipath to handle with a constrained (size and power) receiver in a smartphone.
        • Re:Multipath (Score:4, Informative)

          by tibit ( 1762298 ) on Friday March 02, 2012 @04:13PM (#39224195)

          Hah, CDMA should be plenty robust to multipath, you can use more than one adaptive correlator per channel, and each correlator gives you the relative multipath phase as a diagnostic output, too. Ideally you'd want more than one antenna to make the adaptive scheme more robust, but it'll work with just one. What's more, you can always record the high-bandwidth datastream from the digital radio I/Q inputs for offline data recovery: whatever processing you do online is limited by the maximum latency allowed in your decoder, offline has no such problem. To integrate "offline" with other recording equipment, you can simply have two outputs: a realtime output that goes to the program mixing console, and a more delayed offline output that goes to the multitrack "source" recorder for a studio mix (where you can easily shift things around, time-wise).

          Admittedly satellite links have stable channel properties, but the error correction codes that they use are as close to optimal for given datarate-to-bandwidth as is feasible, and that's not very common in non-cellular consumer point-to-point gear. I agree that terrestrial channels are more challenging when it comes to varying channel properties.

          A wireless mike is a very specific application. It shouldn't ever need a receiver, so the only way to deal with potentially strong narrowband interference is to use CDMA and as wide of a transmit bandwidth as possible -- using frequency division (one channel per mike) is not robust enough, usually. All of the "brains" need to be in the base station, the transmitter circuitry can be hardwired in a relatively simple FPGA that takes input from an audio codec, a couple jumper settings (node ID / code selection) and pushes it via a DAC to the filter/upconverter/final amp. All the encoding etc. is done completely digitally and can be probably modeled in a few pages of Verilog.

    • http://en.wikipedia.org/wiki/Single-frequency_network [wikipedia.org]

      AFAIK however its only used for digital transmission where you can do a lot of signal processing. I don't think it would work well with analog - look what happens on AM at night.

      However SFNs are used with the DAB digital radio system in europe.

      • http://en.wikipedia.org/wiki/Single-frequency_network [wikipedia.org]

        AFAIK however its only used for digital transmission where you can do a lot of signal processing. I don't think it would work well with analog - look what happens on AM at night.

        However SFNs are used with the DAB digital radio system in europe.

        All transmissions are inherently analog in nature. The phrase "digital" only refers to the processing before and after (companding etc). AM refers to a modulation scheme. FM also refers to a modulation scheme. FM is more reliable and has better consistency than AM, however both are analog technologies.

        • by Viol8 ( 599362 )

          Well yes, obviously digital has to be converted to an analog format at some point , but in this case its the type of data being transmitted that matters. Besides which, digital transmission rarely uses AM and uses FM in a far more complicated way that analog radio. There is also phase modulation.

        • by Trinn ( 523103 )

          ...ever heard of a photon? :-P

      • As I explained above, this is different. What you mention share the same channel, while here it adds new independent channels. A closer analogy, although it's different, would be MIMO spatial multiplexing. This said, if it doesn't work with multipath it won't change your life.
      • Actually, when it doesn't involve hearing a bunch of stations at once, the effect of signal combining from multiple paths off the ionosphere can be something of an audio adventure called selective fading. Although I suspect little is written about it, as one of the more unusual parts of the pop culture of a.m. radio I think it deserves a small place in history. (It still happens, No doubt some experienced it when listening to legends such as Wolfman Jack broadcast from outside the U.S. (Unfortunately the

    • oh yeah, first!

      Mod parent Insightful!

  • Re: (Score:2, Interesting)

    Comment removed based on user account deletion
    • Surely the "without using polarization" part means it isn't that?

    • Re:Not really new (Score:5, Insightful)

      by virgil_disgr4ce ( 909068 ) on Friday March 02, 2012 @10:13AM (#39219861) Homepage
      Try reading the article. The innovation is to use orbital angular momentum, NOT spin angular momentum (polarization).
      • Em waves don't HAVE orbital angular momentum.

        • Probably sounds insane to cite, but Rodin coils put out em fields with orbital angular momentum - I've measured it personally from them.
        • Re:not really (Score:4, Informative)

          by jo_ham ( 604554 ) <joham999 AT gmail DOT com> on Friday March 02, 2012 @10:41AM (#39220029)

          Photons do.

          Photons are part of the EM spectrum.

          • Some random fixed point in space? The antenna? What?

            • Re: (Score:3, Informative)

              by Anonymous Coward

              Around the propagation direction of the beam. Read this:

              https://en.wikipedia.org/wiki/Light_orbital_angular_momentum

            • by Baloroth ( 2370816 ) on Friday March 02, 2012 @10:59AM (#39220149)
              See Wikipedia [wikipedia.org] for details. It isn't polarization, but I can't exactly explain how it isn't.
            • by tibit ( 1762298 ) on Friday March 02, 2012 @11:02AM (#39220177)

              The notion of "what are they orbiting" is nonsensical here -- we're talking about quantum objects. It's like saying that electrons "orbit" the nucleus: in the description of their motion, the concept of a classical "path" doesn't quite apply either, and classical mechanics can't describe what an electron does when bound to the nucleus! Now, Maxwell's theory is "classical" in a way, but it describes AFAIK an aggregate (macroscopic) behavior of inherently non-classical, quantum objects, the photons. To get the behavior at the quantum level right, you need quantum electrodynamics (QED).

              It is well known from Maxwell's theory that electromagnetic radiation carries both energy and momentum. The momentum may have both linear and angular contributions; angular momentum has a spin part associated with polarization and an orbital part associated with spatial distribution

              - from "Orbital angular momentum of light and the transformation of Laguerre-Gaussian laser modes" by Allen et al. In the same paper, you can read that you can measure those properties of light using fairly simple opto-mechanical instruments:

              A suspended lambda/2 birefringent plate undergoes torque in transforming right-handed into left-handed circularly polarized light. Suspended cylindrical lenses undergo torque in transforming a Laguerre-Gaussian mode of orbital angular momentum -l*hbar per photon, into one with +I*hbar per photon.

          • Photons are the EM spectrum, you probably meant to say. :)

          • Photons do.

            Photons are part of the EM spectrum.

            Polarization comprises photon angular momentum.

            And there's nothing "orbital" about their apparatus. It's a reflector that imparts polarization at the cost of directivity.

      • I'm not following this. How does one generate such a signal, and receive it, and if it realy works, why demonstrate with only two signals?

        Simple cross polarization people can see with polarized sun glasses. And it's used in LCD displays when a plastic sheet allows light at one polarization, and the liquid crystal can flip its polarization when exposed to a field, the light passing through when it agrees with the filter, blocked when at 90 degrees. If the angles aren't quite right there's bleed through.

        • If I'm understanding this correctly, there is no polarization. The receiving antenna is getting it's voltage changes from the amount of spin energy in the beam and it's location in space, sort of like the way a QAM signal uses phase shift and amplitude changes to create a waveform that wouldn't be possible with either method alone.

          • With a QAM signal, the ability to have one signal cancel allows for a second, but no more. And QAM really isn't doing anything extraordinary. With AM FM or phase modulation, the sidebands produced are mirror images, redundant. So the signal took twice the needed bandwidth to start with. Even analog t.v. from more than 60 years ago managed to filter off most of the lower sideband at the transmitter. They could have done more, but allowed for slop in simple receivers.

            I've studied the links, and it's comp

      • It could mean anything from plain circular polarization (which has been used for ages) to whatever.
        I'm not sure what kind of problem it should solve. After all MIMO system can separate different sources of radio just as well.

        Besides there are always frauds out there, often those don't even understand what they are doing.

      • Reading a bullshit article about a bullshit demonstration won't improve anybody's understanding. The apparatus they show doesn't impart any novel characteristic to EM waves. It's a kind of polarizer and there is nothing new about using linear or circular polarization to transmit independent signals in the same bandwidth.

        They can't use this method to increase the number of signals that can be transmitted in the same bandwidth over conventional methods because they are actually using conventional methods b

    • Please RTFA:
      • this is something new, so by definition this cannot have anything to share with ham radio.
      • TFA talks about angular momentum of the EM field. This has nothing to do with polarization, but with another property of the EM field. Since they are emitting waves with an angular momentum, they cannot be plane wave (it can be shown, see for example here [amazon.com] for details) that plane waves have the angular momentum of the field equal to zero).
      • From the paper it is unclear how do they realize a field with an ang
  • As I said in the other, non-annointed article on this subject:

    I'm not an EM genius, but this sounds an awful lot like circular polarization [wikipedia.org] with perhaps a selectable 'twist' rate. I'd love to see a 3D diagram of a vortex wave vs a circularly polarized wave propagating - that would help me understand what's happening.

    • Polarization is not spatially twisted -- that is just a visualization using the electric field strength on one spatial axis and the magnetic field strength on the second spatial axis, while the radiation travels in the third (real) spatial axis.

      Here is a wikipedia article: https://en.wikipedia.org/wiki/Light_orbital_angular_momentum [wikipedia.org]

      • Thanks for the pointer - that helps. My concept of 'twist rate' seems to be borne out as |m|, although what's shown seems to be the whole EM wave spiraling vs the E and M components being polarized. That's cool!

        The images in the right column of the diagram look suspiciously like propagation modes in circular waveguide [gstatic.com] to me.

        You might also be interested in this ham's work which exploits the Aharonov-Bohm Effect [wikipedia.org]:

        Robert Zimmerman, NP4B/VE3RKZ, describes five years of research at McMaster University in “

  • by neyla ( 2455118 ) on Friday March 02, 2012 @10:10AM (#39219829)

    This might help, but it doesn't expel Shannon-Hartley. They don't get "inifinite channels" in finite bandwith. Not unless each channel has infinitely low capacity, anyway.

    • Re: (Score:2, Funny)

      Does an infinite no. of channels with infinitely low capacity give infinite effective bandwidth?
      • "Infinitely low" is zero. So no.
      • Depends on what the limit of the product of the no. of channels by the capacity of the channel becomes as they approach infinity. Calculus, basically: one thing can approach zero and another can approach infinity, but their product can approach a finite number in between. It all depends on how they approach zero and infinity. Could also mean the bandwidth is effectively infinite, or effectively zero, in the extreme case, although the latter would be unexpected, to say the least.

    • by Idbar ( 1034346 ) on Friday March 02, 2012 @10:25AM (#39219937)
      I think AT&T is fine with that. ;-)
    • by tibit ( 1762298 )

      Shannon's theory applies to an abstract concept of a channel. It says nothing about how you map such an abstract channel to a physical realization of it. So, you cannot make a leap from an abstract channel and abstract bandwidth to a physical realization using some means of transmission without saying how those concepts map to underlying physical reality. Do that first, otherwise your statement makes no sense.

    • This might help, but it doesn't expel Shannon-Hartley. They don't get "inifinite channels" in finite bandwith. Not unless each channel has infinitely low capacity, anyway.

      The other limiting factor in Shannon-Hartley is signal power. Transmitting with infinite power does allow you to have infinite channel capacity, and transmitting over an infinite number of channels each with finite power over does just that. That said, I am sure that practical limitations in hardware design will place a limit on how close the orbital angular momentum spacing can be and still be able to discriminate the channels.

      • by pavon ( 30274 )

        Oh and I should add; the reason we don't solve the current spectrum congestion problem by just boosting signal power is because one channel's signal is another channel's noise unless filters can be designed to separate them. So without improvements in filters having everyone boost their signal power will just mean that everyone's noise floor increases as well and the whole thing is a wash as far as channel capacity goes (except now we are wasting more power).

        I would imagine that transmitting 10 channel with

    • It doesn't invalidate Shannon, as an AC says below and as I understand it, it creates new orthogonal channels. Think MIMO spatial multiplexing.

      But as the first poster said, and IIUC from how it's done, it shouldn't be robust to multipath. If true this would limit a possibly application to microwave trunking, but wouldn't help WiFi of your smartphone.
    • by fritsd ( 924429 )
      Maybe you'd need an infinite number of carefully placed small spiky antennas to get infinite channels.
      Seems like they are doing something like the interferometry of the extremely boring-looking LOFAR [wikipedia.org] radiotelescope.
      Amirite?
      • by neyla ( 2455118 )

        Better antennas is equivalent to a lower noise-floor, so yes, better antennas do genuinely allow you to transmit more data, especially if they are directional since that then also raises the effective transmit-power. (what matters is the power/noise ratio)

  • by neBelcnU ( 663059 ) on Friday March 02, 2012 @10:20AM (#39219905) Journal

    "It's potentially a boon for congested spectrum problems, although at the moment I suspect it would only work for directional links."

    Wouldn't that mean a huge boon for telcos and state gov'ts that still use terrestrial microwave links? Could a state network take advantage of this, and sell off the unused portion? Speaking for IL and MN, both have microwave line-of-sight to all their toll booths, truck depots and weigh stations.

    There are inevitably issues to this, but if this first appears in LoS, wouldn't these networks (telco+local gov't) be able to use it?

    • I was thinking of the DVD player in my minivan. It's one of those devices that broadcasts on at some FM frequency and I have to tune the car radio to it. It's pretty much unusable any time I pass through NYC, which, since I live on Long Island, is basically any road trip long enough to bother with putting a DVD on.

  • by Anonymous Coward

    I think that what's happening is no different from what you could achieve with a 802.11/n MIMO system. Think of their twisted antenna as a ring of patch antennas.

    Essentially, the trade-off they are making is that they broaden the beam by warping their antenna, so they have a lower-gain antenna with a wider beam. Consequently, you need more power in each of the two orbital angular momentum states to transmit the data, consequently Shannon-Hartley is preserved.

    Another way of looking at it is that their di

    • I'm not sure if that is completely correct
      802.11/n MIMO uses a crude form of adaptive beamforming, where you screw with phasing to make intentional nulls in your antennas' receive/transmit pattern (this is also used in EW to null out jammers)

      This doesn't seem like the same thing, although I'd be lying if I said I completely understood the article
    • Yep. In layman's terms, imagine a row of many dish antennas sending data to an identical row of dish receivers. You can send a lot more data than just one dish, right? But think about the problems: as you try to make the dishes smaller, the signal beams spread out and overlap multiple antennas, causing interference and crosstalk. Same happens if the dishes aren't aligned exactly right, or the signals bounce off walls on the way.

      Wrap the whole thing around in a circle and apply a little math hocus pocus,

  • BULLSH!T (Score:4, Interesting)

    by AB3A ( 192265 ) on Friday March 02, 2012 @10:43AM (#39220037) Homepage Journal

    Any time someone starts talking of infinite channel capacity, you know they're going to be full of crap. Shannon's limit is a Mathematical principle. There is no such thing as "infinite" bandwidth/channel capacity.

    What they're actually discussing is the spatial equivalent of spread spectrum. In other words, they have their own custom reflector with its own unique shape that can be reversed so that a coherent signal with minimal inter-symbol interference would be present. It is not a bad idea, except that you would need a line of sight path with very little exposure to the first Fresnel zones. Reflections would be a bitch to deal with.

    Also note this method reduced point source noise, but it doesn't eliminate it. Likewise, a spread spectrum signal is still detectable as increased noise in a narrow-band radio.

    • by tibit ( 1762298 )

      They are talking about a physical channel, not an abstract channel. Shannon's limit as you've properly said is a mathematical concept. You have to map it to a physical reality in a particular way. It may well be that such a mapping has some hitherto unused possibilities, and that's precisely what the authors are saying.

    • *In principle* is a key modifier, so let's think this through. Doesn't the EMR spectrum *in principle* have infinite capacity if we could use arbitrarily high frequencies to transmit information? I suppose there may be some modern-physics-predicted upper limit to what frequencies of radiation can exist, just as I suppose there may be a limit also to the orbital angular momentum states, but that's not what's at issue with violation of Shannon's limit. Shannon's limit applies to one-dimensional signal. I'm su
      • by Prune ( 557140 )
        Infinite frequency is impossible even in principle because it implies infinite energy per photon (-> infinite mass -> black hole with event horizon at infinity), so, NO. This was such an obvious point that I'm surprised you put your foot in your mouth with that comment like this.
        • I believe you missed my point. I admitted that there's likely some reason for an upper limit (and no, IANAP), that's not what at issue with violation of Shannon's limit as per OP, which is what I was addressing. Even still, doesn't your argument about infinite energy still leave us unbounded? It does not provide a strict upper limit to what frequencies we can manipulate, and therefore, still infinite in principle. In practice, obviously, we can't expect to keep upping the energy without bound, I don't and d
          • by Prune ( 557140 )
            You're wrong again: there is an upper bound placed on us by cosmological considerations: the accelerating expansion of the universe means that our Hubble volume will forever only contain a finite amount of mass-energy (eventually everything outside the gravitationally bound local group of galaxies will be carried away from us by expansion at speed greater than the speed of light, making it forever out of reach). So where the hell are you going to get the arbitrary amount of energy you need to make an arbitr
            • I believe you missed my point. I admitted that there's likely some reason for an upper limit (and no, IANAP), that's not what at issue with violation of Shannon's limit as per OP, which is what I was addressing.

              I'm glad you can sort-of derive that upper limit. Good for you. You know far more physics than I.

              Maybe you and I just have different interpretations of what "in principle" means in this context. I'm only referring to the point at hand, which was violation of a the mathematical principle. How we realize the application of that mathematical principle in the physical world is subject to whatever limitations due to our choice of medium. That the medium of EMR has the limitations you point out is wonderful,

    • Re:BULLSH!T (Score:5, Interesting)

      by Angst Badger ( 8636 ) on Friday March 02, 2012 @11:31AM (#39220411)

      Shannon's limit is a Mathematical principle.

      Unfortunately, most people have next to no understanding of mathematics beyond some rote memorization from school. This is just another example of people confusing analog signals with magic. To be fair, the actual researchers involved probably understand this quite well, but the scientifically uneducated class from which science and technology journalists are drawn is another matter.

      The non-mathematical version, for those interested, is that yes, analog signals are continuous and so can occupy an infinite number of states. The reason you can't get infinite bandwidth out of that is because both the transmitters and receivers have limited precision, and because there is always noise, which is another manifestation of the Second Law. For example, there are an infinite number of real numbers between 0 and 1. If you could actually use all of that space, you could encode any amount of information in an arbitrarily short signal. (Well, there's a limit to that, too, for which see Georg Cantor.) In practice, you can't use all of that space, because your instruments might distinguish quite well between 0.001 and 0.002, but they can't reliably tell the difference between 0.001 and 0.0005. On top of that, there is noise, which is also a big topic, but you can think of it as a random fluctuation in the signal. If the ambient noise varies between 0.0 and 0.0005 in the same example, you can't even reliably tell the difference between 0.001 and 0.002.

      What the parent is getting at is that laws of physics, being derived from observations of nature with limited precision, might occasionally be overturned by better observations. Fundamental mathematical principles, on the other hand, are much more reliable. There might be a difference between rest mass and inertial mass that we could exploit for thrustless propulsion. It's extremely unlikely, but it can't be ruled out. But there is zero possibility that 2 + 2 will ever equal anything other than four. Shannon's limit and, for that matter, the Nyquist sampling theorem are a little more complex than a simple integer sum, but the actual math for both would fit on an index card with plenty of room to spare to blather on about "infinite" analog signals. We use digital signals most of the time these days because it makes the hardware easier to design, but neither digital nor analog can be used to make an end run around the Second Law.

      What the researchers in TFA claim to have figured out is another way to use part of the signal outside of the frequency domain to stuff data into. It's a really ingenious approach that might be quite useful if it pans out in actual practice, but it's not magic, and it's not infinite.

      • by Prune ( 557140 )
        Infinite bandwidth would also violate the Bekenstein bound and holographic principle. But hey, people will still think they can encode arbitrary precision real numbers in physical objects with finite extent, as futile as that dream is.
    • Indeed it is bullshit.
      They are talking of using a different basis to send the energy ...e.g Sine waves is a commonly used mathematical basis.
      So they are using different functions here and they are saying they will be able pack more energy in ...they have not shown that at all
      and the fact they are talking "infinite" capacity...tells me there is more "you know what" that real proof here.

  • by Urban Garlic ( 447282 ) on Friday March 02, 2012 @10:44AM (#39220049)

    I RT first part of the FA (no, not actually new here...), and an important point is that the paper is talking about *orbital* angular momentum of the light beam. The circular polarization states correspond to *spin* angular momentum of the photons, orbital angular momentum is a different thing with its own phase space.

    Infinite channels still seems unlikely, it has to be true that detectors for orbitally-tuned light beams won't be perfect, and will detect "nearby" orbitally-tuned beams as well, and it's likely that some parts of the space of orbital angular momentum will be more difficult to generate than others, so I remain skeptical of the claim.

    But, the mechanism is not a trivial one. I note with some surprise that TFS actually correctly notes that it's orbital angular momentum they're talking about.

    • by mprinkey ( 1434 )

      I think the key take-away is that there is another physical signal dimension to exploit--frequency, directionality, polarization, and now orbital angular momentum. They have demonstrated that they can distinguish between two channels on the same frequency using orbital angular momentum as the differentiator. So, OAM mode can be added to the tool kit. If they can distinguish among a few dozen modes and still allow beam forming, this could provide a huge benefit for cellular and other wireless networks. I

  • Each and every advance in RF technology, the final signal becomes more and more chaotic. The entropy rises and rises as we cram more and more data into the available frequencies.

    Also, the broadcasts have to become more directional and they use less energy.

    Anyways, it's pretty easy to extrapolate on this trend. What will the RF emissions from earth look like in 1000 years, when we've developed radio technology to the physical limits? I suspect that those signals will be completely indistinguishable from n

    • But 1000 years from now, observers in a planet 1000 light-years away will be receiving our current transmissions, in their very "old and inefficient" modulations.

      • But 1000 years from now, observers in a planet 1000 light-years away will be receiving our current transmissions, in their very "old and inefficient" modulations.

        Unfortunately, those observers will be busy working on how to make usable tools out of bronze and so our signals will pass right by. By the time they develop radio technology 3000 years later, we will either have blown ourselves into oblivion or be communicating using far more advanced technology as the parent comment suggested.

        • And what exactly makes you think they will be less advanced than us? I mean it could possibly be, I'm just curious if there's a reason or not. Because what I first thought was that anyone else would likely be more advanced than us.

          • Oh I don't. Just pointing out how narrow the window is given the vastness of space and time.

            Will the SETI project continue for another 3000 years? What if we're using quantum entanglement to communicate at faster than light speeds, would we even notice if a weak electromagnetic signal wandered into the neighborhood? Completely different communications gear, probably not.

      • by sjames ( 1099 )

        If their technology is sufficiently advanced. Otherwise by the time they develop good enough receivers, our signals will look like noise.

        It doesn't preclude detection, but it does considerably reduce the odds.

  • NOTHING NEW - PROVEN (Score:3, Informative)

    by OveE ( 2587057 ) on Friday March 02, 2012 @11:31AM (#39220409)
    The claims made by Thidé et al. about finding an entirely new mechanism that can improve wireless communication, as reported by BBC in "'Twisted' waves could boost capacity of wi-fi and TV" (http://www.bbc.co.uk/news/science-environment-17221490), have been proven incorrect in the following peer reviewed journal paper: "O. Edfors, A. J. Johansson: Is orbital angular momentum (OAM) based radio communication an unexploited area? IEEE Transactions on Antennas and Propagation, Vol. 60, No. 2, pp. 1126-1131, 2012." Existing and well known techniques produce the same 'twisted' radio waves. These 'twisted' waves bring nothing conceptually new in the area of wireless communications and cannot boost capacity further. The claims have been appearing repeatedly in media over the last few years, while consensus among experts in the area of wireless communications is that they are incorrect.
    • by Anonymous Coward

      The manuscript version of the paper mentioned above can be downloaded here:
      http://lup.lub.lu.se/luur/download?func=downloadFile&recordOId=2062936&fileOId=2339120

  • by colsandurz45 ( 1314477 ) on Friday March 02, 2012 @11:32AM (#39220421)

    I did my MS thesis on wideband spectrum sensing (just about everything under 2.2 GHz). Turns out the spectrum isn't actually overcrowded, it's underutilized, especially over 500 MHz. Look at some papers by the Shared Spectrum Company www.sharedspectrum.com/. This is common misperception and it's the result of FCC policies (that they're working on changing). The underlying problem is that institutions that have spectrum allocated for them now actually need it, just not most of time.

  • by Guppy ( 12314 ) on Friday March 02, 2012 @11:42AM (#39220513)

    "...There are simple tricks that are almost never noticed till a very high technology is attained. For instance, quantum torsion antennas can be built from silver and cobalt steel arrays, if the geometry is correct. Unfortunately, finding the proper geometry involves lots of theory and the ability to solve some large partial differential equations. There are many Slow Zoners who never discover the principle."

  • Sounds like this might be useful for coaxial or fiber optic cable as well, perhaps vastly increasing the capacity of that? It might be able to be put to use in a much more quick way on that medium. Then you dont have to worry about the electromagnetic waves breaking strands of DNA in your body since its confined to the cable, given the very well justified concerns that EMF waves from cell phones could damage the body, which is an entirely reasonable concern since DNA in the body is actually rather delicate

  • The same as MIMO (Score:4, Informative)

    by Jott42 ( 702470 ) on Friday March 02, 2012 @12:31PM (#39221103)
    This is actually a subset of MIMO, which is already widely used in WiFi and other wireless networks. Thus it will, regrettably, not give access to any additional bandwidth. The details on the equivalence is in a paper from IEEE Transactions on Antennas and Propagation, titled "Is orbital angular momentum (OAM) based radio communication an unexploited area?" http://lup.lub.lu.se/luur/download?func=downloadFile&recordOId=2062936&fileOId=2339120 [lub.lu.se]
    • Yes. It's easy to get lost in the weeds when talking about rotational systems, especially when light is involved. Here's an analogue for what I think is going on:

      Suppose I had a rectangular grid of directional transmitting antennas, and a rectangular grid of directional receivers. If I point one antenna array at the other, I can send data between each pair. With enough antennas and receivers, I can send arbitrarily large amounts of data using a fixed bandwidth. But there are problems: if I don't have r

  • by MajroMax ( 112652 ) on Friday March 02, 2012 @12:51PM (#39221343)
    I am a scientist, but not an E&M specialist. Take this with a grain of salt.

    I've read through the New Journal of Physics article. The ``radio vorticity'' means that the phase of the signal goes through a 180 flip across the beam centre, and the zero-point of this phase shift rotates as you move along the beam. The receiving antennas in the experiment were a pair of yagis, used to create a radio interferometer. The math and experimental results behind this appaer sound, but there are a few limitations:

    • This is a highly directional effect. Not only would multipath interference destroy the crap out of this signal, but they also needed pairs of antennas on opposite sides of the beam centre to discriminate between mode-0 and mode-1 rotations. Directionally-wide beams will have more interference, and building the interferometer will be more difficult with less than a 180 separation.
    • The transmitting antenna was very specialized. The transmitter itself not so much, but the antenna was a parabolic antenna ``mechanically modified'' -- they sliced through the top of it to turn the atenna into one loop of a parabolic spiral. If you have access to the article online, take a look at the picture, it's kind of neat.
    • ``In principle an infinite number of channels'' my ass. They're building an interferometer, so they need at least one antenna per mode they wish to discriminate between, and when they used antenna-separation to do the phase filtering for them they saw some significant interference form secondary lobes for intervals where the match wasn't perfect. This was okay for the two-channel experiment (mode 0 and 1), but the receiving antenna design would really start messing with higher channels, where those secondary lobes start seriously interfering themselves.
    • As written, the receiving antenna design is highly sensitive. The phase cancellation used required some pretty precise antenna positioning, since they needed a displacement of one half-wavelength in the beam direction for proper interference (to discriminate the mode 1 angular momentum). Trying this in a production environment is going to be pretty tricky -- perhaps they could get somewhere with electronic phase delay.

    So for controlled channels -- perhaps even microwave links -- I'm optimistic about engineers being able to build something useful out of this. But the basic math isn't going to generalize to omnidirectional links, and it certainly isn't going to deal well with strong multipath interference. Simply being able to discriminate between modes requires straddling the beam centre, so this absolutely isn't going to work for general consumption.

    Also, I don't think that practical antenna design will ever allow more than three or four channels of angular momentum outside of a lab setting. Even that may potentially be a huge win for fixed microwave links, though.

  • by Anonymous Coward

    The reference paper wishes that it was describing a fundamentally newmultiplexing technique. However, it is not. They are instead using the well-known technique of antenna cancellation to exploit spatial diversity to create additional orthogonal channels. This is fine, but it requires that your antenna be larger than a quarter wavelength (so that it can have nulls) and in fact much larger still so that the nulls in your radiation pattern do not steer all of your transmitted power 90 degrees away from you

  • "physicists have shown that it is possible to use two beams of incoherent radio waves"

    Finally! A use for Rush Limbaugh!

  • It turns out that if you can use circularly polarized light to manipulate nanoscopic objects, and make them spin.

    I wonder what would happen if you built a 2 Ghz transmitter system which fed an antenna with a spin on the order of 10? Would it cause something like the Norway Spiral by spinning the air that happened to absorb part of the signal?

    Google "Orbital Angular Momentum" and you'll find all sorts of stuff that almost seems like magic, or science fiction, that actually could work.

  • It took a few hours contemplation for me to finally get my mind wrapped around this one, here's my explanation.

    Imagine you have an AC motor, with 2 poles. If driven with 60 hz power, it will spin at 3600 rpm (60 rotations per second).
    The same 60 hz power can be fed into a 4 pole AC motor which will spin at 1800 rpm (30 rotations per second)
    Generalizing, The same 60 hz power can be used for a 2n pole motor to get a spin of (7200 /.n) rotations per second.

    If you were to hold a magnet in the center of any of

"If it ain't broke, don't fix it." - Bert Lantz

Working...